Which one of the following, if substituted for the condition that Lewis and Ota do not lecture on the same day as eac...

Bola on October 4 at 08:00PM

November 2018 SEC 2 QX 6

Why is B incorrect, if K&O lecture same day KOJ LMN , we would still be having JK on the same day which is the same effect that as if we have L we can not have O on the same day.

Reply
Create a free account to read and take part in forum discussions.

Already have an account? log in

Emil-Kunkin on October 6 at 04:33PM

Hi, B would ban scenarios that were previously allowed. We previously could have had

JKL
MNO,

But b would ban this, so it would not have the same effect as the initial rule.